Quantcast
  • Register
PhysicsOverflow is a next-generation academic platform for physicists and astronomers, including a community peer review system and a postgraduate-level discussion forum analogous to MathOverflow.

Welcome to PhysicsOverflow! PhysicsOverflow is an open platform for community peer review and graduate-level Physics discussion.

Please help promote PhysicsOverflow ads elsewhere if you like it.

News

PO is now at the Physics Department of Bielefeld University!

New printer friendly PO pages!

Migration to Bielefeld University was successful!

Please vote for this year's PhysicsOverflow ads!

Please do help out in categorising submissions. Submit a paper to PhysicsOverflow!

... see more

Tools for paper authors

Submit paper
Claim Paper Authorship

Tools for SE users

Search User
Reclaim SE Account
Request Account Merger
Nativise imported posts
Claim post (deleted users)
Import SE post

Users whose questions have been imported from Physics Stack Exchange, Theoretical Physics Stack Exchange, or any other Stack Exchange site are kindly requested to reclaim their account and not to register as a new user.

Public \(\beta\) tools

Report a bug with a feature
Request a new functionality
404 page design
Send feedback

Attributions

(propose a free ad)

Site Statistics

205 submissions , 163 unreviewed
5,047 questions , 2,200 unanswered
5,345 answers , 22,709 comments
1,470 users with positive rep
816 active unimported users
More ...

  Conformal symmetry with Goldston bosons

+ 3 like - 0 dislike
495 views

I am looking for examples of the physical systems with both conformal symmetry and Goldston bosons. The systems can be in any physical context, like QCD phases, Condensed Matter (Graphene, etc), or SUSY gauge theories.

Naively, in 1+1D gapless system, we can more easily have conformal symmetry. But to have a spontaneous symmetry breaking with Goldston bosons, and together with a (perhaps emergent) conformal symmetry seem to be somehow more challenging.

In 2+1D and higher dimensional system, we may find harder to have conformal symmetry. Are there such examples of conformal symmetry with Goldston bosons.?

This post imported from StackExchange Physics at 2020-10-28 19:06 (UTC), posted by SE-user annie marie heart
asked Apr 28, 2017 in Theoretical Physics by annie marie heart (1,205 points) [ no revision ]
You won't have Goldstone modes in 1+1D because of the Mermin-Wagner theorem (unless you're at the large central charge limit).

This post imported from StackExchange Physics at 2020-10-28 19:06 (UTC), posted by SE-user user106422
You can't have unbroken conformal symmetry and GBs because their decay constant is dimensionfull in d>2. Even more physically, a theory of GB has a cutoff and its therefore not scale invariant. The only way out I know is to break conformal invariance spontaneously too, with the appearance of a dilaton.

This post imported from StackExchange Physics at 2020-10-28 19:06 (UTC), posted by SE-user TwoBs

1 Answer

+ 3 like - 0 dislike

You can't have unbroken conformal symmetry and Goldstone Bosons (GB)'s at the same time because the GB decay constant $f_\pi$ is dimensionfull in $d>2$. And in $d=2$ there are no (physical) GB's. Even more physically, a theory of GB has a cutoff $\Lambda$ and its therefore not scale invariant.

The only way out to this is by weakening the requests, e.g. allowing for a non-linearly realized conformal invariance. In this case, conformal symmetry is broken spontaneously too, and a light dilaton appears in the spectrum alongside with the GB's from breaking spontaneously a global internal continuous symmetry. The leading terms in the effective action (ind $d=4$) for a dilaton $\sigma$ and ordinary GBs $\pi$ would take the form $$ \mathcal{S}[\sigma,\pi]=\int d^4x \frac{f^2_\sigma}{2}(\partial_\mu e^{\sigma})^2+ \frac{f_\pi^2}{2} e^{2\sigma}(\partial_\mu\pi)+\ldots $$ which is scale invariant under $x\rightarrow x e^{\alpha}$ if the dilaton transforms non-linearly $$ \sigma(x)\rightarrow \sigma(x e^{\alpha})+ \alpha\,,\qquad \pi(x)\rightarrow \pi(x e^{\alpha})\,. $$ The best way to see how this invariance is realized is by actually making a field redefinition $$\chi\equiv f_\sigma e^{\sigma/f_\sigma}$$ so that the action becomes $$ S=\int d^4x \frac{1}{2}(\partial_\mu\chi)^2+\frac{1}{2}\left(\frac{f_\pi}{f_\sigma}\right)^2\chi^2(\partial_\mu\pi)^2 $$ which contains only ratio of scales $f_\pi/f_\sigma$ and it's hence scale invariant. The physical scales are recovered because the new $\chi$ variables has non-vanishing vev that breaks spontaneously conformal invariance $$ \langle\chi\rangle= f_\sigma\,. $$

This post imported from StackExchange Physics at 2020-10-28 19:06 (UTC), posted by SE-user TwoBs
answered Apr 30, 2017 by TwoBs (315 points) [ no revision ]

Your answer

Please use answers only to (at least partly) answer questions. To comment, discuss, or ask for clarification, leave a comment instead.
To mask links under text, please type your text, highlight it, and click the "link" button. You can then enter your link URL.
Please consult the FAQ for as to how to format your post.
This is the answer box; if you want to write a comment instead, please use the 'add comment' button.
Live preview (may slow down editor)   Preview
Your name to display (optional):
Privacy: Your email address will only be used for sending these notifications.
Anti-spam verification:
If you are a human please identify the position of the character covered by the symbol $\varnothing$ in the following word:
p$\hbar$ysicsOve$\varnothing$flow
Then drag the red bullet below over the corresponding character of our banner. When you drop it there, the bullet changes to green (on slow internet connections after a few seconds).
Please complete the anti-spam verification




user contributions licensed under cc by-sa 3.0 with attribution required

Your rights
...